Control Systems Test Questions Set 3

1)   Which system exhibits the initiation of corrective action only after the output gets affected?

a. Feed forward
b. Feedback
c. Both a and b
d. None of the above
Answer  Explanation  Related Ques

ANSWER: Feedback

Explanation:
No explanation is available for this question!


2)   What should be the nature of bandwidth for a good control system?

a. Large
b. Small
c. Medium
d. All of the above
Answer  Explanation  Related Ques

ANSWER: Large

Explanation:
No explanation is available for this question!


3)   If an impulse response of a system is e-5t, what would be its transfer function?

a. 1/ s - 5
b. 1/ s + 5
c. (s+1) / (s+5)
d. (s2 - 5s)/ (s-5)
Answer  Explanation  Related Ques

ANSWER: 1/ s + 5

Explanation:
No explanation is available for this question!


4)   Which among the following are the elements of rotational motion?

a. Mass, Spring, Friction
b. Inertia, Damper, Spring
c. Work, Energy, Power
d. Force, Pressure, Viscocity
Answer  Explanation  Related Ques

ANSWER: Inertia, Damper, Spring

Explanation:
No explanation is available for this question!


5)   Match the following notations with their meanings:

A. G(s) ---------- 1) Laplace of error signal
B. H(s) ---------- 2) Laplace of output signal
C. C(s) ---------- 3) Forward transfer function
D. E(s) ---------- 4) Feedback transfer function


a. A- 2, B- 3, C- 1, D- 4
b. A- 3, B- 4, C- 2, D- 1
c. A- 2, B- 3, C- 4, D- 1
d. A- 1, B- 2, C- 3, D- 4
Answer  Explanation  Related Ques

ANSWER: A- 3, B- 4, C- 2, D- 1

Explanation:
No explanation is available for this question!


6)   At summing point, more than one signal can be added or _________

a. Subtracted
b. Multiplied
c. Both a and b
d. None of the above
Answer  Explanation  Related Ques

ANSWER: Subtracted

Explanation:
No explanation is available for this question!


7)   The value of variables at each node is _________the algebraic sum of all signals arriving at that node.

a. Less than
b. Equal to
c. Greater than
d. None of the above
Answer  Explanation  Related Ques

ANSWER: Equal to

Explanation:
No explanation is available for this question!


8)   In signal flow graph, the product of all ______gains while going through a forward path is known as 'Path gain'.

a. Branch
b. Path
c. Node
d. Loop
Answer  Explanation  Related Ques

ANSWER: Branch

Explanation:
No explanation is available for this question!


9)   If a type 1 system is subjected to parabolic input, what will be the value of steady state error?

a. 0
b. 100
c. Constant k
d. Infinite
Answer  Explanation  Related Ques

ANSWER: Infinite

Explanation:
No explanation is available for this question!


10)   On which factor does the steady state error of the system depend?

a. Order
b. Type
c. Size
d. Prototype
Answer  Explanation  Related Ques

ANSWER: Type

Explanation:
No explanation is available for this question!


11)   Consider the equation S3 + 3s2 + 5s + 2 = 0. How many roots are located in left half of s-plane?

a. Zero
b. Two
c. Three
d. Four
Answer  Explanation  Related Ques

ANSWER: Three

Explanation:
No explanation is available for this question!


12)   If the system is represented by characteristic equation s6 + s4 + s3 + s2 + s + 3 = 0, then the system is_________

a. Stable
b. Unstable
c. Marginally stable
d. Unpredictable
Answer  Explanation  Related Ques

ANSWER: Unstable

Explanation:
No explanation is available for this question!


13)   If poles are added to the system, where will the system tend to shift the root locus?

a. To the left of an imaginary axis
b. To the right of an imaginary axis
c. At the center
d. No shifting takes place
Answer  Explanation  Related Ques

ANSWER: To the right of an imaginary axis

Explanation:
No explanation is available for this question!


14)   For a unity feedback system with G(s) = 10 / s2, what would be the value of centroid?

a. 0
b. 2
c. 5
d. 10
Answer  Explanation  Related Ques

ANSWER: 0

Explanation:
No explanation is available for this question!


15)   If 'ξ' approaches to zero, the peak resonance would ________

a. Also be zero
b. Be unity
c. Tend to infinity
d. Become equal to peak overshoot
Answer  Explanation  Related Ques

ANSWER: Tend to infinity

Explanation:
No explanation is available for this question!


16)   At which condition of 'ξ', resonant peak does not exist and its maximum value is considered to be unity along with zero resonant frequency?

a. 0 < ξ < 0.707
b. ξ > 0.707
c. ξ = 0
d. ξ = 1
Answer  Explanation  Related Ques

ANSWER: ξ > 0.707

Explanation:
No explanation is available for this question!


17)   If the damping of the system becomes equal to zero, which condition of the resonant frequency is likely to occur?

a. ωr = ωd
b. ωr > ωn
c. ωr < ωn
d. ωr = ωn
Answer  Explanation  Related Ques

ANSWER: ωr = ωn

Explanation:
No explanation is available for this question!


18)   If the resonant peak is estimated to be '5', which among the following would be the correct value of damping?

a. ξ = 0.3
b. ξ = 1
c. ξ = 3.2
d. ξ = 5.55
Answer  Explanation  Related Ques

ANSWER: ξ = 0.3

Explanation:
No explanation is available for this question!


19)   If a system is said to have a damping ξ = 0.5532 with the natural frequency ωn = 2 rad/sec, what will be the value of resonant frequency (ωr)?

a. 1.2456 rad/s
b. 1.7352 rad/s
c. 2.3421 rad/s
d. 3.66 rad/s
Answer  Explanation  Related Ques

ANSWER: 1.2456 rad/s

Explanation:
No explanation is available for this question!


20)   In frequency response, the resonance frequency is basically a measure of  _______ of response.

a. Speed
b. Distance
c. Angle
d. Curvature
Answer  Explanation  Related Ques

ANSWER: Speed

Explanation:
No explanation is available for this question!


21)   The frequency at which the phase of the system acquires ____ is known as 'Phase crossover frequency'.

a. 90°
b. -90°
c. 180°
d. -180°
Answer  Explanation  Related Ques

ANSWER: -180°

Explanation:
No explanation is available for this question!


22)   At which frequency does the magnitude of the system becomes zero dB?

a. Resonant frequency
b. Cut-off frequency
c. Gain crossover frequency
d. Phase crossover frequency
Answer  Explanation  Related Ques

ANSWER: Gain crossover frequency

Explanation:
No explanation is available for this question!


23)   If the phase angle at gain crossover frequency is estimated to be -105°, what will be the value of phase margin of the system?

a. 23°
b. 45°
c. 60°
d. 75°
Answer  Explanation  Related Ques

ANSWER: 75°

Explanation:
No explanation is available for this question!


24)   The system is said to be marginally stable, if gain margin is ______

a. 0
b. 1
c. +∞
d. None of the above
Answer  Explanation  Related Ques

ANSWER: +∞

Explanation:
No explanation is available for this question!


25)   If the constant 'k' is positive, then what would be its contribution on the phase plot?

a.
b. 45°
c. 90°
d. 180°
Answer  Explanation  Related Ques

ANSWER: 0°

Explanation:
No explanation is available for this question!


26)   If the unity feedback system is given by the open loop transfer function G(s) = ks2 / [(1 + 0.3s) (1+ 0.05s)], what would be the initial slope of magnitude plot?

a. 20 dB/decade
b. 40 dB/decade
c. 60 dB/decade
d. Unpredictable
Answer  Explanation  Related Ques

ANSWER: 40 dB/decade

Explanation:
No explanation is available for this question!


27)   If the system is represented by G(s) H(s) = k (s+7) / s (s +3) (s + 2), what would be its magnitude at ω = ∞?

a. 0
b.
c. 7/10
d. 21
Answer  Explanation  Related Ques

ANSWER: 0

Explanation:
No explanation is available for this question!


28)   According to Nyquist stability criterion, where should be the position of all zeros of q(s) corresponding to s-plane?

a. On left half
b. At the center
c. On right half
d. Random
Answer  Explanation  Related Ques

ANSWER: On left half

Explanation:
No explanation is available for this question!


29)   Consider a feedback system with gain margin of about 30. At what point does Nyquist plot crosses negative real axis?

a. -3
b. -0.3
c. -30
d. -0.03
Answer  Explanation  Related Ques

ANSWER: -0.3

Explanation:
No explanation is available for this question!


30)   For Nyquist contour, the size of radius is _______

a. 25
b. 0
c. 1
d.
Answer  Explanation  Related Ques

ANSWER: ∞

Explanation:
No explanation is available for this question!


31)   If a Nyquist plot of G (jω) H (jω) for a closed loop system passes through (-2, j0) point in GH plane, what would be the value of gain margin of the system in dB?

a. 0 dB
b. 2.0201 dB
c. 4 dB
d. 6.0205 dB
Answer  Explanation  Related Ques

ANSWER: 6.0205 dB

Explanation:
No explanation is available for this question!


32)   Which principle specifies the relationship between enclosure of poles & zeros by s-plane contour and the encirclement of origin by q(s) plane contour?

a. Argument
b. Agreement
c. Assessment
d. Assortment
Answer  Explanation  Related Ques

ANSWER: Argument

Explanation:
No explanation is available for this question!


33)   Which among the following constitute the state model of a system in addition to state equations?

a. Input equations
b. Output equations
c. State trajectory
d. State vector
Answer  Explanation  Related Ques

ANSWER: Output equations

Explanation:
No explanation is available for this question!


34)   State model representation is possible using _________

a. Physical variables
b. Phase variables
c. Canonical state variables
d. All of the above
Answer  Explanation  Related Ques

ANSWER: All of the above

Explanation:
No explanation is available for this question!


35)   Which mechanism in control engineering implies an ability to measure the state by taking measurements at output?

a. Controllability
b. Observability
c. Differentiability
d. Adaptability
Answer  Explanation  Related Ques

ANSWER: Observability

Explanation:
No explanation is available for this question!


36)   According to the property of state transition method, e0 is equal to _____

a. I
b. A
c. e-At
d. -eAt
Answer  Explanation  Related Ques

ANSWER: I

Explanation:
No explanation is available for this question!


37)   Which among the following is a disadvantage of modern control theory?

a. Implementation of optimal design
b. Transfer function can also be defined for different initial conditions
c. Analysis of all systems take place
d. Necessity of computational work
Answer  Explanation  Related Ques

ANSWER: Necessity of computational work

Explanation:
No explanation is available for this question!


38)   Which among the following is a unique model of a system?

a. Transfer function
b. State variable
c. Both a and b
d. None of the above
Answer  Explanation  Related Ques

ANSWER: Transfer function

Explanation:
No explanation is available for this question!


39)   Which architectural unit/block of PLC decides the sequence of different operations to be executed by means of instructions written in memory?

a. Memory
b. Programming software
c. I/O interface
d. CPU
Answer  Explanation  Related Ques

ANSWER: CPU

Explanation:
No explanation is available for this question!


40)   Which among the following units of PLC is adopted to convey the control plan to CPU?

a. Memory
b. Power supply unit
c. I/O interface
d. Programming software
Answer  Explanation  Related Ques

ANSWER: Programming software

Explanation:
No explanation is available for this question!